Can someone help me wit this please

Can Someone Help Me Wit This Please

Answers

Answer 1

Answer:

Hi

Please mark brainliest ❣️

Can Someone Help Me Wit This Please

Related Questions

546, 400 and 4,856 The value of 4 in which number is how many times larger than the value of 4 in which number.​

Answers

To determine how many times larger the value of 4 is in the second number compared to the first number, we need to calculate the ratio of the values.

First number: 546
Second number: 4,856

In the first number, the value of 4 is the same as the digit itself since it appears once.

In the second number, the value of 4 is larger since it appears twice.

To find the ratio, we divide the value of 4 in the second number by the value of 4 in the first number:

Value of 4 in second number: 2
Value of 4 in first number: 1

Ratio: 2/1 = 2

Therefore, the value of 4 in the second number is two times larger than the value of 4 in the first number.

You purchase a tarp to cover the driveway when it snows. The
dimensions of your driveway are 10.2 ft. by 15.7 ft. If the tarp covers
your entire driveway, how many square feet are covered? Your answer
should be a number only. Do not round.

Answers

If the dimensions of your driveway are 10.2 ft. by 15.7 ft and the tarp covers your entire driveway,  the square feet are covered is [tex]160.14ft^{2}[/tex]

How can the dimension be calculated?

In mathematics, a dimension is the length or width of an area, region, or space in one direction. It is just the measurement of an object's length, width, and height.

With the given conditions,  we can formulate the expression as

;10.2 ft. * 15.7 ft

=160.14

Learn more about dimensions at;

https://brainly.com/question/26740257

#SPJ1

The amount of time a certain brand of light bulb lasts is normally distributed with a mean of 2000 hours and a standard deviation of 25 hours. Out of 665 freshly installed light bulbs in a new large building, how many would be expected to last between 2030 hours and 2060 hours, to the nearest whole number?

Answers

To determine the number of light bulbs expected to last between 2030 hours and 2060 hours, we need to calculate the z-scores corresponding to these values and then use the z-score formula to find the proportion of light bulbs within this range.

The z-score formula is given by:

z = (x - μ) / σ

where:

x = value

μ = mean

σ = standard deviation

For 2030 hours:

z1 = (2030 - 2000) / 25

For 2060 hours:

z2 = (2060 - 2000) / 25

Now, we can use the z-scores to find the proportions associated with each value using a standard normal distribution table or calculator. The table or calculator will provide the area/proportion under the normal curve between the mean and each z-score.

Let's calculate the z-scores and find the proportions:

z1 = (2030 - 2000) / 25 = 1.2

z2 = (2060 - 2000) / 25 = 2.4

Using a standard normal distribution table or calculator, we can find the proportions corresponding to these z-scores:

P(z < 1.2) ≈ 0.8849

P(z < 2.4) ≈ 0.9918

To find the proportion of light bulbs expected to last between 2030 hours and 2060 hours, we subtract the cumulative probabilities:

P(2030 < x < 2060) = P(z1 < z < z2) = P(z < z2) - P(z < z1)

P(2030 < x < 2060) ≈ 0.9918 - 0.8849

Finally, we multiply this proportion by the total number of light bulbs (665) to get the estimated number of light bulbs expected to last between 2030 hours and 2060 hours:

Number of light bulbs ≈ (0.9918 - 0.8849) * 665

Rounding to the nearest whole number, the expected number of light bulbs that would last between 2030 hours and 2060 hours is approximately 71.[tex]\huge{\mathfrak{\colorbox{black}{\textcolor{lime}{I\:hope\:this\:helps\:!\:\:}}}}[/tex]

♥️ [tex]\large{\textcolor{red}{\underline{\mathcal{SUMIT\:\:ROY\:\:(:\:\:}}}}[/tex]

Find the measure of ∠F
.

Answers

Step-by-step explanation:

triangle EFG is an isosceles triangle

angle G

= 180°-58°

= 122° (adj. angles on a str. line)

angle F

= (180°-122°)÷2

= 29° (angles in a triangle)

QUESTION 1 1.1 1.2 1.4 Use the definition of the derivative (first principles) to determine f'(x) if f(x)=2x 1.3 Determine f'(x) from first principles if f(x)=9-x². Determine f'(x) from first principles if f(x)=-4x².​

Answers

Based on the functions given, it should be noted that the values will be 2, -2x and -8x.

How to calculate the value

Using the definition of the derivative, we have:

f'(x) = lim(h->0) [f(x + h) - f(x)] / h

= lim(h->0) [2(x + h) - 2x] / h

= lim(h->0) 2h / h

= lim(h->0) 2

= 2

Therefore, f'(x) = 2.

For f(x) = 9 - x²:

Using the definition of the derivative, we have:

f'(x) = lim(h->0) [f(x + h) - f(x)] / h

= lim(h->0) [9 - (x + h)² - (9 - x²)] / h

= lim(h->0) [9 - (x² + 2xh + h²) - 9 + x²] / h

= lim(h->0) [-2xh - h²] / h

= lim(h->0) (-2x - h)

= -2x

Therefore, f'(x) = -2x.

For f(x) = -4x²:

Using the definition of the derivative, we have:

f'(x) = lim(h->0) [f(x + h) - f(x)] / h

= lim(h->0) [-4(x + h)² - (-4x²)] / h

= lim(h->0) [-4(x² + 2xh + h²) + 4x²] / h

= lim(h->0) [-4x² - 8xh - 4h² + 4x²] / h

= lim(h->0) [-8xh - 4h²] / h

= lim(h->0) (-8x - 4h)

= -8x

Therefore, f'(x) = -8x.

Learn more about functions on

https://brainly.com/question/31878183

#SPJ1

Suppose there are 17 jelly beans in a box-2 red, 3 blue, 4 white, and 8 green. What part of the jelly beans is blue? As a decimal rounded to the nearest ten-thousandth (four decimal places)

Answers

Blue Jelly beans are 0.1764 part of total .

Given,

Total beans = 17

Blue = 3

Red =2

White =4

Green =8

Now,

Out of total , green jelly beans = 8/17

Out of total , red jelly beans = 2/17

Out of total , white jelly beans = 4/17

Out of total , blue jelly beans = 3/17

Hence the blue jelly beans are 0.1764 part of total jelly beans .

Know more about decimal,

https://brainly.com/question/8985071

#SPJ1

6 I need steps to know how we did it

Answers

Answer:

D

Step-by-step explanation:

the right triangle contains h , the horizontal leg and the sloping side which is the hypotenuse of the right triangle.

the horizontal leg is half the measure of the side of the square base.

horizontal leg = 8 ÷ 2 = 4

using Pythagoras' identity in the right triangle

the square on the hypotenuse is equal to the sum of the squares on the other 2 sides, that is

h² + 4² = 10² ( subtract 4² from both sides )

h² = 10² - 4² ( take square root of both sides )

h = [tex]\sqrt{10^2-4^2}[/tex]

Factorizing Trinomials in the form: x² + bx + c 3.1 x² + bx + c Find two integers, r and s, whose product is c and whose sum is b to rewrite the trinomial as: x² + rx + sx + c Factorizing x² + 5x + 6 3.1.1 What is the value of b and c in the trinomial? b = C = ACTIVITY 3 3.1.2 Use the table below to determine the two integers, r and s. Factors of 6 1 and 6 -1 and and 3 2 and 3 6 Product of the two Sum of the two factors factors 1+6=7 1+-=-7 2+3=5 --21-3=-5 1x6-6 -1X-6=6 2x3 = 6 -2x-3-6 product Result 6 but sur Which two integers will correctly provide the values of b and c in the express x2 + 5x + 6? 1.3 Rewite x² + 5x + 6 as an equivalent expression in the form x² + -4 Use the knowledge obtained from activity 2 on grouping and the dis to factorize the expression.​

Answers

The values of b and c is 5 and 6.

To factorize the trinomial x² + 5x + 6, we need to find two integers whose product is 6 and whose sum is 5.

From the given table, we can see that the integers 2 and 3 satisfy these conditions.

Therefore, we can rewrite the trinomial as:

x² + 5x + 6 = x² + 2x + 3x + 6

x² + 2x + 3x + 6 = (x² + 2x) + (3x + 6)

Now, we can factor out the common terms from each group:

x² + 2x + 3x + 6 = x(x + 2) + 3(x + 2)

                          = (x + 2)(x + 3)

Therefore, the factored form of the trinomial x² + 5x + 6 is (x + 2)(x + 3).

Regarding the values of b and c, we can see that b = 5 and c = 6 in the trinomial x² + 5x + 6.

Learn more about Polynomial here:

https://brainly.com/question/11536910

#SPJ1

I NEED HELP WITH STATISTICS

Answers

Am here for you so need anything don’t message me

Round to the nearest given place.
1.45169 thousandths

Answers

Answer:

1.452

Step-by-step explanation:

1.45169 rounded to the thousandths place would be 1.452

Example: Divide 3 loaves between 5 people First, divide two of the loaves into thirds... each person gets one third each, with one third left over Then divide the left-over third from the second loaf into fifths So, each person gets: 1/5 and the third loaf into fifths each person gets one fifth each each person gets a slice (one fifteenth) 1/15 3/5 The Egyptians used the approximated process to work on the area of a circle as shown in the picture. 1.4 Show the representation of the fractions on the second row. (2) 1.5 Show the algorithm/abstract strategy to justify the 3/5 found as the answer. (3)​

Answers

The algorithm justifies the answer of 3/5 as the fraction each person gets.

Representation of the fractions on the second row:

From what you described, two of the loaves were divided into thirds.

This means each person receives one third, and there is one third remaining. Then, this remaining third from the second loaf was further divided into fifths.

Therefore, each person receives one fifth from this remaining third.

So, the representation of the fractions on the second row would be:

Each person receives 1/3 (one third) from the two loaves.

Each person receives 1/5 (one fifth) from the remaining third.

Algorithm/Abstract strategy to justify the 3/5 found as the answer:

To find the final answer of 3/5, we can follow the steps you provided:

Divide two loaves into thirds, giving each person 1/3.

Divide the remaining third from the second loaf into fifths, giving each person 1/5.

Combining the fractions, each person has 1/3 + 1/5.

To add these fractions, we need to find a common denominator. In this case, the least common multiple (LCM) of 3 and 5 is 15. We can convert 1/3 and 1/5 to have a denominator of 15:

1/3 = 5/15 (multiplying numerator and denominator by 5)

1/5 = 3/15 (multiplying numerator and denominator by 3)

Now, we can add the fractions:

5/15 + 3/15 = 8/15

Therefore, each person receives 8/15 of a loaf.

Simplifying this fraction, we get 3/5.

Hence, the algorithm justifies the answer of 3/5 as the fraction each person gets.

Learn more about Algorithm/Abstract strategy click;

https://brainly.com/question/29438718

#SPJ1

Jessica needs to know how much water her new fish tank can hold:

A rectangular prism with a length of 8 inches, a width of 4 inches, and a height of 9 inches.

Determine the total volume of the fish tank.

Answers

The fish tank has a total volume of 288 inch³. As a result, Jessica's new fish tank has a capacity of 288 inch³ for water.

The volume of a rectangular prism can be calculated using the formula:

V = l x b x h..........(i)

where,

V ⇒ Volume

l  ⇒ length

b ⇒ width

h ⇒ height

From the question, we are given the values,

l = 8 inches

b = 4 inches

h = 9 inches

Putting these values in equation (i), we get,

V = 8 x 4 x 9

⇒ V = 288 in³

Therefore, the fish tank has a total volume of 288 inch³. As a result, Jessica's new fish tank has a capacity of 288 inch³ for water.

Learn more about the volume of rectangular prism on:

https://brainly.com/question/24284033

please help! mathematicians

Answers

Answer:

1 < m < 4

Step-by-step explanation:

If the roots of function f(x) are not real, then the discriminant (the part under the square root sign) will be negative.

Set the discriminant less than zero and rewrite in standard form:

[tex]\begin{aligned}16-4m(-m+5)& < 0\\16+4m^2-20m& < 0\\4m^2-20m+16& < 0\\4(m^2-5m+4)& < 0\\m^2-5m+4& < 0\end{aligned}[/tex]

Factor the quadratic:

[tex]\begin{aligned}m^2-5m+4& < 0\\m^2-4m-m+4& < 0\\m(m-4)-1(m-4)& < 0\\(m-1)(m-4)& < 0\end{aligned}[/tex]

The leading coefficient of the quadratic m² - 5m + 4 is positive.

Therefore, the graph will be a parabola that opens upwards.

This means that the interval where the parabola is below the x-axis (negative) is between the zeros of the quadratic. Since the zeros are m = 1 and m = 4, the solution to the inequality is 1 < m < 4.

Therefore, the values of m for which the roots of function f(x) will be non-real are 1 < m < 4.

I need the solution!!!!​

Answers

Solve for the first variable in one of the equations, then substitute the result into the other equation.

Point form :
(-4,0)

Equation form :
x = -4, y = 0

(q11) Find the center of mass of the system of objects that have masses 2 , 3 and 5 at the point (-1,2),(1,1) and (3,3) respectively.

Answers

The center of mass of the system is approximately (3.7, 2.6).

The center of mass of a system of objects is the point where all the weight of the system appears to be concentrated. It can be defined as the average location of the weighted parts of the system.

The center of mass of a system is dependent on the mass of the objects in the system and their positions.

Let's determine the center of mass of the system with masses of 2, 3, and 5 at the points (-1, 2), (1, 1), and (3, 3), respectively. Let's name the masses m1, m2, and m3, respectively, and the coordinates (x1, y1), (x2, y2), and (x3, y3).

The x-component of the center of mass is given by the formula:

x= (m1x1 + m2x2 + m3x3) / (m1 + m2 + m3)

The y-component of the center of mass is given by the formula:

y= (m1y1 + m2y2 + m3y3) / (m1 + m2 + m3)

By using the given values, let's calculate the x and y components of the center of mass:

x = (2 x -1 + 3 x 1 + 5 x 3) / (2 + 3 + 5) = 37/10 ≈ 3.7y

= (2 x 2 + 3 x 1 + 5 x 3) / (2 + 3 + 5)

= 26/10 = 2.6

To learn more about : mass

https://brainly.com/question/28916233

#SPJ8

The number of combinations of eight items taken three at a time can be written as

Answers

Answer: 8C3

Step-by-step explanation: You need to use Combinations for this. Out of 8, you need to select 3, so answer is 8C3.

Multiply three consecutive digits backwards starting from 8, and divide by 3 factorial

(8*7*6)/(3*2*1)

=56

Edwin sells jars of jam for $1.90 each. Determine how many jars of jam Edwin needs to sell to break even if the variable cost per jar is $1.10 and fixed expenses are $35,700.00 per year.

Answers

Edwin needs to sell 44,625 jars of jam to break even.

To determine how many jars of jam Edwin needs to sell to break even, we'll calculate the breakeven point using the following formula:

Breakeven Point = Fixed Expenses / (Selling Price per Unit - Variable Cost per Unit)

Given information:

Selling Price per Unit (SP) = $1.90

Variable Cost per Unit (VC) = $1.10

Fixed Expenses = $35,700.00 per year

Plugging in the values into the formula:

Breakeven Point = $35,700 / ($1.90 - $1.10)

Breakeven Point = $35,700 / $0.80

Breakeven Point = 44,625 jars

Therefore, Edwin needs to sell 44,625 jars of jam to break even.

for such more question on breakeven point

https://brainly.com/question/30551452

#SPJ8

what is the greatest common factor of 97 and 24? what the answer

Answers

1

Because the number 97 is a prime number

Answer:

The greatest common factor (GCF) of two numbers is the largest number that divides evenly into both numbers. Since 97 is a prime number and 24 is not divisible by 97, the GCF of 97 and 24 is 1.

Which is the equation of the given line in point-slope form?

y−0=−1(x−8)

y−0=1(x+8)

y=−x+8

y−8=−1(x+0)

Answers

Answer:

y = -x + 8

Step-by-step explanation:

Let's break down the equation step by step to understand it better.

The equation in point-slope form is given as:

y - y1 = m(x - x1)

In this case, we have:

y - 0 = -1(x - 8)

The point-slope form uses a specific point (x1, y1) on the line and the slope (m) of the line.

Here, the point (x1, y1) is (8, 0), which represents a point on the line. This means that when x = 8, y = 0. The graph has a point at (8, 0), which confirms this information.

The slope (m) is -1 in this equation. The slope represents the rate at which y changes with respect to x. In this case, since the slope is -1, it means that for every unit increase in x, y decreases by 1. The negative sign indicates that the line has a downward slope.

By substituting the values into the equation, we get:

y - 0 = -1(x - 8)

Simplifying further:

y = -x + 8

This is the final equation of the line in slope-intercept form. It tells us that y is equal to -x plus 8. In other words, the line decreases by 1 unit in the y-direction for every 1 unit increase in the x-direction, and it intersects the y-axis at the point (0, 8).

If the graph has points at (0, 8) and (8, 0), the equation y = -x + 8 accurately represents that line.

How do you solve the question Deloitte signs a contract on December 1 to provide 40 days of advisory services with receipt of $20,000 due at the end of the contract. On December 31, 75% of the services have been completed.

Answers

As of December 31, Deloitte should recognize $15,000 as revenue for the advisory services completed.

To solve the given question, we need to determine the amount of revenue that Deloitte should recognize as of December 31, based on the percentage of services completed.

Here's how we can calculate it:

Calculate the total revenue for the contract:

Total revenue = $20,000

Determine the percentage of services completed:

Percentage of services completed = 75%

Calculate the revenue recognized as of December 31:

Revenue recognized = Percentage of services completed × Total revenue

= 75% × $20,000

= $15,000

Therefore, as of December 31, Deloitte should recognize $15,000 as revenue for the advisory services completed.

Learn more about revenue click;

https://brainly.com/question/29567732

#SPJ1

Problem
Find the equation of the line.
Use exact numbers.

Answers

The Equation of line is y= -3/2x + 60

From the graph we take two coordinates as (2, 0) and (0, 3)

We know the formula for slope

Slope= (Change in y)/ (Change in x)

Slope = (3-0)/ (0-2)

Slope= 3 / (-2)

Slope= -3/2

Now, Equation of line

y - 0 = -3/2 (x-  2)

y= -3/2x + 6

Thus, the Equation of line is y= -3/2x + 60.

Learn more about Slope here:

https://brainly.com/question/3605446

#SPJ1

A scientist mixes water (containing no salt) with a solution that contains 35% salt. She wants to obtain 140 ounces of a mixture that is 15% salt. How many
ounces of water and how many ounces of the 35% salt solution should she use?

Answers

Answer:

.35x = 140(.15)

.35x = 21

x = 60 oz of 35% salt.

The scientist will need 60 oz of the 35% salt solution and 80 oz of water.

Express 75 as a product of its prime factors write the prime factors in ascending order and give your answer in index form

Answers

Step-by-step explanation:

75 = 3 x 5 x 5    in prime factorization

Answer:

Step-by-step explanation:

3x5x5


Minka pours 1/4 cup of milk on her oatmeal each day for 7

Answers

Assuming you want the amount of milk in 7 days, we can set up a multiplication problem. Given Minka pours 1/4 cup of milk in her oatmeal each day, we can multiply that by 7 days to find that:

1/4 = 0.25
0.25 • 7 = 1.75, or 1 3/4

By the end of 7 days, Minka pours 1 3/4 cups of milk into her oatmeal collectively.

Determine the a) total annual cost, and b) cost per mile to the nearest cent.
1. Liz Nolan drove 34,500 miles last year. The total of fixed costs was $9,916 and of variable costs was
$4,897.

Answers

Answer:

total annual cost: 49313

cost per mile: 14 cents

Step-by-step explanation:

find total annual cost by adding everything up

find cost per mile by doing 4897/34500

cost/ miles

we use variable cost since the only thing that might change each year is the amount of miles they drive

fixed costs are fixed and don't change

I NEED HELP WITH STATISTICS

Answers

(a) The null hypothesis is that the mean birth weight of babies born at full term is 7.2 pounds. The alternative hypothesis is that the mean birth weight of babies born at full term is greater than 7.2 pounds.

(b) If the scientist decides to reject the null hypothesis, she might be making a Type I error.

(c) A Type II error occurs when the null hypothesis is false, but the scientist fails to reject it.

How to explain the information

a A Type I error occurs when the null hypothesis is true, but the scientist rejects it. In this case, the null hypothesis is that the mean birth weight of babies born at full term is 7.2 pounds. If the scientist rejects this hypothesis, she is saying that she believes that the mean birth weight is greater than 7.2 pounds. However, if the null hypothesis is true, then the mean birth weight is actually 7.2 pounds, and the scientist has made a mistake.

b In this case, the scientist would fail to reject the null hypothesis and conclude that the mean birth weight of babies born at full term is 7.2 pounds. However, the true mean birth weight is 7.7 pounds, so the scientist would be making a Type II error.

c In the context of a Type II error, suppose the null hypothesis is false, meaning there is indeed a significant difference or relationship. However, due to various factors such as insufficient sample size, low statistical power, or other limitations, the scientist fails to reject the null hypothesis. Consequently, they accept the null hypothesis even though it is false, leading to a Type II error.

Learn more about hypothesis on

https://brainly.com/question/606806

#SPJ1

line
A storage bin has the shape of a cylinder with a conical top. What is the volume of the storage bin if
its radius is r = 4.9 ft, the height of the cylindrical portion is h = 9.7 ft, and the overall height is
H = 16.3 ft?
Volume (to the nearest tenth)

Answers

Answer:

Step-by-step explanation:

To find the volume of the storage bin, we need to calculate the volumes of both the cylindrical portion and the conical top, and then add them together.

The volume of the cylindrical portion can be calculated using the formula:

V_cylinder = π * r^2 * h

where r is the radius and h is the height of the cylindrical portion.

Substituting the given values, we have:

V_cylinder = π * (4.9 ft)^2 * 9.7 ftV_cylinder ≈ 748.07 ft³ (rounded to two decimal places)

The volume of the conical top can be calculated using the formula:

V_cone = (1/3) * π * r^2 * H_cone

where r is the radius and H_cone is the height of the conical top.

The height of the conical top can be obtained by subtracting the height of the cylindrical portion from the overall height:

H_cone = H - h = 16.3 ft - 9.7 ft = 6.6 ft

Substituting the given values, we have:

V_cone = (1/3) * π * (4.9 ft)^2 * 6.6 ftV_cone ≈ 243.24 ft³ (rounded to two decimal places)

To find the total volume, we add the volume of the cylindrical portion and the volume of the conical top:

Total volume = V_cylinder + V_cone

Total volume ≈ 748.07 ft³ + 243.24 ft³

Total volume ≈ 991.31 ft³ (rounded to one decimal place)

Therefore, the volume of the storage bin is approximately 991.3 ft³ (rounded to the nearest tenth).

Thus the required volume is, 975.05  ft³

Given that,

radius = r = 4.9

Height of cylindrical potion = h = 9.7

Overall height = 16.3

Since,

total height = Height of the cylinder + height of the cone

Height of the cone = 16.3 - 9.7

                                = 6.6 m

Since we know that,

Volume of a cylinder = πr² h

⇒ π (4.9)²(9.7)

⇒ 731.29 ft³

Since we also know that

Volume of a cone = (1/3)πr² h

= 731.29/3

= 243.76  ft³

Volume of the bin = volume of cone + volume of cylinder

= 731.29 ft³  + 243.76  ft³

Hence the volume be,

= 975.05  ft³

To learn more about volume visit:

https://brainly.com/question/16860802

#SPJ1

'Two containers designed to hold water are side by side, both in the shape of a cylinder. Container A has a diameter of 12 feet and a height of 14 feet. Container B has a diameter of 10 feet and a height of 20 feet. Container A is full of water and the water is pumped into Container B until Container B is completely full.To the nearest tenth, what is the percent of Container A that is full after the pumping

Answers

The nearest tenth, approximately 93.5% of Container A is full after the water is pumped into Container B.

To determine the percentage of Container A that is full after the water is pumped into Container B, we need to compare the volumes of the two containers.

The volume of a cylinder can be calculated using the formula: V = πr^2h, where V is the volume, π is a constant (approximately 3.14159), r is the radius, and h is the height.

For Container A:

Radius (r) = Diameter / 2 = 12 ft / 2 = 6 ft

Height (h) = 14 ft

For Container B:

Radius (r) = Diameter / 2 = 10 ft / 2 = 5 ft

Height (h) = 20 ft

Now, let's calculate the volumes of the two containers:

Volume of Container A = π * (6 ft)^2 * 14 ft ≈ 1,679.65 ft^3

Volume of Container B = π * (5 ft)^2 * 20 ft ≈ 1,570.8 ft^3

To find the percentage of Container A that is full, we need to calculate the ratio of the volume of water in Container B to the volume of Container A:

Ratio = Volume of Container B / Volume of Container A

Ratio = 1,570.8 ft^3 / 1,679.65 ft^3 ≈ 0.9347

Finally, to convert this ratio to a percentage, we multiply it by 100:

Percentage = Ratio * 100

Percentage ≈ 0.9347 * 100 ≈ 93.5%

Therefore, to the nearest tenth, approximately 93.5% of Container A is full after the water is pumped into Container B.

for such more question on percentage

https://brainly.com/question/24877689

#SPJ8

22% of what number is 3300

Answers

To find the number that corresponds to 22% of a given value, you can divide the given value by 22% (or 0.22).

Let's use this approach to find the number:

3300 ÷ 0.22 = 15,000

So, 22% of 15,000 is equal to 3300.

Answer:

x = 15000

Step-by-step explanation:

If you are using a calculator, simply enter 3300×100÷22, which will give you the answer.

3) Last year the mean salary for professors in a particular community college was $62,000 with a standard deviation of $2000. A new two year contract is negotiated. In the first year of the contract, each professor receives a $1500 raise.

Find the mean and standard deviation for the first year of the contract.
b) In the second year of the contract, each professor receives a 3% raise based on their salary during the first year of the contract. Find the mean and the standard deviation for the second year of the contract.

Answers

a) Mean for the first year of the contract: $63,500

The standard deviation for the first year of the contract: $2,000.

b) Mean for the second year of the contract: $65,405.

The standard deviation for the second year of the contract: $60.

We have,

To find the mean and standard deviation for the first year of the contract, we can use the given information and the properties of the normal distribution.

Given:

The mean salary for professors in the previous year = $62,000

Standard deviation in the previous year = $2,000

Raise in the first year = $1,500

Mean for the first year of the contract:

The mean salary for the first year can be obtained by adding the raise to the previous mean:

Mean = Previous Mean + Raise

Mean = $62,000 + $1,500

Mean = $63,500

The standard deviation for the first year of the contract:

Since each professor receives the same raise, the standard deviation remains the same:

Standard Deviation = $2,000

Therefore, for the first year of the contract, the mean salary is $63,500, and the standard deviation remains $2,000.

Now,

In the second year of the contract, each professor receives a 3% raise based on their salary during the first year of the contract.

To find the mean and standard deviation for the second year, we can use the given information and the properties of the normal distribution.

Mean for the second year of the contract:

To calculate the mean for the second year, we need to add a 3% raise to the mean salary of the first year:

Mean = Mean of the first year + (3% * Mean of the first year)

Mean = $63,500 + (0.03 * $63,500)

Mean = $63,500 + $1,905

Mean = $65,405

The standard deviation for the second year of the contract:

Since each professor receives a raise based on their salary from the first year, the standard deviation also increases. To calculate the standard deviation, we multiply the standard deviation from the first year by the percentage increase:

Standard Deviation = Standard Deviation of the first year * (Percentage Increase / 100)

Standard Deviation = $2,000 * (3 / 100)

Standard Deviation = $2,000 * 0.03

Standard Deviation = $60

Therefore, for the second year of the contract, the mean salary is $65,405, and the standard deviation is $60.

Thus,

a) Mean for the first year of the contract: $63,500

The standard deviation for the first year of the contract: $2,000.

b) Mean for the second year of the contract: $65,405.

The standard deviation for the second year of the contract: $60.

Learn more about mean here:

https://brainly.com/question/23263573

#SPJ1

Other Questions
Write the equation of a circle with the given center and radius. center = (4, 9), radius = 4___ Solve the equation. e^(13x-1) = (e11)^x If you were testing whether the difference between two population means were equal (thus your H 0 : 1 = 2 ), what value would you find in the center of your randomization distribution? a. The sample mean difference. b. The value of the sample mean for group 1. c. The value of the sample mean for group 2 . d. approximately 0 ABC is an Australian firm. Assume the following: 1. ABC needs 100M HKD for 12 months 2. After 12 months, ABC wishes to sell the HKD 3. The firm currently does not hold any HKD assets 4. The firm is risk-averse and wishes to minimize transaction costs 5. The current spot rate is EAUD/HKD=0.22 What should be ABCs strategy?A. Enter into a swap with a counterparty: ABC pays 22M AUD to receive 100M HKD now. After 12 months, ABC sells 100M HKD for 21.8M AUD. B. Purchase 100M HKD at EAUD/HKD =0.22 now. Enter into a forward contract now to sell 100M HKD after 12 months at FAUD/HKD=0.215 C. Purchase 100M HKD at EAUD/HKD =0.22 now. After 12 months, sell 100M HKD at the prevailing spot exchange rate. D. Strategies mentioned in A) and B) are equally as attractive E. None of the above Reflect on a time you have seen language interfere with a message either at work, school or in your personal life. How might a change in the formality, simplicity, or reduction in redundancy have made the message easier to understand? If you have never experienced a situation like this, simply reflect on how you believe the use of industry jargon, as opposed to direct plain language, might affect the audience's ability to decode and act on a message. Once posted, respond to two or three of your peers. Your response should engage the author of the post by either asking further questions about the scenario, comparing the scenario to a personal experience, or commenting on the action taken in relation to how you would have either behaved similarly or differently in the given situation. Question 2 Discuss the possible products or services that companies engaged in international business can offer to countries with an ageing population. Calculate the change in operating cash flow for a firm for each year using the following information: The asset costs $1,000,000 and is depreciated using MACRS for a 3-year asset. The machine will reduce operating expenses by $120,000 per year for three years. The tax rate is 45%.Working capital needs decrease by $10,000 when the machine is placed in service and are increased at the end of the life of the machine. The asset can be sold for $400,000 at the end of three years. John Dow wants to choose one mutual fund to invest all his money in. There is a set of available mutual funds with similar investment styles and risk profiles that he considers. Which criterion would be the most appropriate for John Dow to make a decision:a. Choose the fund with the highest Sharpe ratiob. Choose the fund with the highest alphac. Choose the fund with the highest information ratiod. Choose the fund with the highest Treynor ratio A car radiator needs a 40% antifreeze solution. The radiator nowholds 20 liters of a 20% solution.How many liters of this should be drained and replaced with 100%antifreeze to get the desiredstren Rentech Ltd company a biotech company, is expected to grow rapidly in the next three years and then have a level growth rate for the foreseeable future. The company expects free cash flows of $327.4 million, $543.3 million, and $843.3 million over the next three years, and thereafter its cash flows will grow at a steady rate of 6 percent per annum. The company has no non-operating assets (NOA). If the appropriate WACC is 9.36 percent, what is the enterprise value of this business?Round the answer to two decimals. If a company receives money in advance of performing a service, the journal entry to record the transaction 1. debits cash and credits prepaid services 2. debits cash and credits accounts receivable 3. debits unearned fees and credits accounts payable 4. debits cash and credits unearned service revenue. Pick the only logical ranking of investment project decision rules listed below (listed from best to worst): (a) IRR, Average Accounting Return Rules, NPV, Payback (b) Payback, IRR, Average Accounting Return Rules, NPV (c) NPV, IRR, Payback, Average Accounting Return Rules (d) NPV, IRR, Average Accounting Return Rules, Payback You finished putting the dishes away after washing them, but neglected to close the cupboards. You reached for your phone on the counter and hit the cupboard with your forehead. Your forehead is now a purplish color and is slightly swollen. What is the treatment for the injury for this scenario? Which of the following words has a prefix meaning no or not? -Infertility -Hypogonadism -Polyuria -Epidural -Reproductive Suppose this morning SPB Ltd., a solar panel and battery manufacturing company, paid a dividend of $3 per share. At 10 a.m. SPB announced its latest earnings results (which were unexpectedly good) and management seems very happy. One year ago, SPB paid for and implemented the Stealth Disposal (SD) project for handling its toxic waste. Only SPB's management is aware of the SD project. The project has reduced SPB's expenses related to toxic waste disposal, but there is a 25% chance of toxic contamination of the drinking water for the nearest city. SPB management has no intention of publicly disclosing that the SD project exists. Following this morning's 10 a.m. quarterly earnings announcement, SPB's earnings growth prospects look very good based upon substantial cost savings (although investors don't know it, these cost saving are due to the SD project). SPB's earnings announcement has caused investors to reassess SPB's dividend growth with the new expectation that dividends will grow by 0.4% each quarter. Prior to the announcement, dividends were expected to grow by 0.3% each quarter. In addition, given investors' perceived risk of SPB, the equity cost of capital for SPB is 11% (effective annual rate). Complete parts a through f below.a. Assume the market is semi strong-form efficient. What was the price of SPB's stock at 9:30 a.m.?b. Continue to assume the market is semi strong-form efficient. What was the price of SPB's stock at 10:30 a.m.?c. By what percent did SPB's stock price change between 9:30 and 10:30 a.m.? Is this consistent with the efficient markets hypothesis? Which word does not have a simillar to energize1. Rejuvenate2. Strengthen3. Enervate 4. Uplift A dividend payout ratio of 75% would be typical ofA.a firm in the rapid growth stage.B.a newly listed company trying to attract investors.C.a profitable firm with limited growth prospects.D.a company whose earnings were also growing at about 75% per year. In the article, "THE FOUNDATIONS OF ENTERPRISE PERFORMANCE:DYNAMIC AND ORDINARY CAPABILITIES IN AN (ECONOMIC) THEORY OF FIRMS"by DAVID J. TEECE:What is the background? Why is the article important? This post is based on the discussion of McClelland's Theory of Needs, in video 2 of the motivation series. Which of McClelland's 3 needs do you think a study revealed that effective managers have a high need for and which of his 3 needs do you think the study found that effective managers have a low need for?Explain your choices, what made you select the high need and what made you select the low need? The same mass of 5 different potential fuels was used to heat the same mass of water in a simple calorimeter. The results are shown below. Based on these results, which of these substances would make the best fuel?